Тёмный

Ideals and Quotient Rings -- Abstract Algebra 19 

Подписаться
Просмотров 5 тыс.
% 148

⭐Support the channel⭐
Patreon: www.patreon.com/michaelpennmath
Merch: teespring.com/stores/michael-penn-math
My amazon shop: www.amazon.com/shop/michaelpenn
🟢 Discord: discord.gg/Ta6PTGtKBm
⭐my other channels⭐
Main Channel: ru-vid.com
non-math podcast: ru-vid.com/show-UCIl9oLq0igolmqLYipO0OBQ
⭐My Links⭐
Personal Website: www.michael-penn.net
Instagram: melp2718
Randolph College Math: www.randolphcollege.edu/mathematics/
Research Gate profile: www.researchgate.net/profile/Michael_Penn5
Google Scholar profile: scholar.google.com/citations?user=W5wkSxcAAAAJ&hl=en

Опубликовано:

 

26 апр 2023

Поделиться:

Ссылка:

Скачать:

Готовим ссылку...

Добавить в:

Мой плейлист
Посмотреть позже
Комментарии : 10   
@karl131058
@karl131058 Год назад
Sorry, but: warmup 3 is wrong. S is not a subring, because it contains 2+i and 2-i (a=b=1 and a=1,b=-1, resp.) but their product is 2^2-i^2, which is 5, NOT an element of S (because of the odd real part), so S is not closed under multiplication
@Hipeter1987
@Hipeter1987 Год назад
Around 19:00, don't we also need the minimum power of a monomial in f*g to be greater than or equal to 2? We have that, but it wasn't mentioned.
@schweinmachtbree1013
@schweinmachtbree1013 Год назад
Yes, this is a mistake - Michael got confused between the degree of a polynomial and the order of a polynomial, the latter being the _least_ power of x with a non-zero coefficient. So the ideal is I = {p(x) ϵ *Z* [x] : ord(p) ≥ 2}. For polynomials over any integral domain (here *Z* ) we have deg(pq) = deg(p) + deg(q), and for polynomials over a general ring we have deg(pq) ≤ deg(p) + deg(q) - likewise for the order we have over any integral domain that ord(pq) = ord(p) + ord(q), and over a general ring we have the inequality ord(pq) ≥ ord(p) + ord(q). There are also inequalities for deg(p+q) and ord(p+q). (these are inequalities even over integral domains; for example over *Z* we have deg(pq) ≤ max(deg(p), deg(q)) where equality may not occur, e.g. deg((x^2 + x) + (1 - x^2)) = deg(x + 1) = 1 < 2 = max(deg(x^2 + x), deg(1 - x^2)).)
@finnr3472
@finnr3472 Год назад
Great series!
@kapoioBCS
@kapoioBCS Год назад
Getting into the serious stuff 😊
@shahrukhshikalgar6714
@shahrukhshikalgar6714 8 месяцев назад
Why while checking for ideal he didn't consider that it must be a subring. A set of polynomials with degree more than equals 2 clearly dosent contain 0 element, hence implies no subring and thus not an ideal!
@schweinmachtbree1013
@schweinmachtbree1013 Год назад
It would have been nice to prove the first part of the proposition at 25:14 for arbitrary intersections, in light of the remark at 32:29 - also for the third part of that proposition we only actually used that I is a left ideal and J is a right ideal; that is, the product of a left ideal and a right ideal is a (two-sided) ideal. Also at 15:56 it would have been good to point out that 0 ϵ I. One small mistake at 34:53 : the well-ordering principle does not mean that subsets of the natural numbers achieve their minima; it says that the minima of subsets of *N* exist - any set with a minimum achieves its minimum (you are thinking of infima, where a set with an infimum _m_ might not achieve _m_ , e.g. (0, 1].)
@Spacexioms
@Spacexioms Год назад
15:34 is the only condition needed to have something be a subring? Quotient rings must have zero divisors?
@fakezpred
@fakezpred Год назад
No, quotient rings may be an integral domain. For example, if R is a ring and I is a prime ideal of R, then R/I is an integral domain
@Spacexioms
@Spacexioms Год назад
@@fakezpred gotcha 👍🏻 thank you!